Use the image below to answer the identify the following. Please show ur work Bc I have too !!


One pair of vertical angles


One pair of adjacent angles


One pair of supplementary angles

Use The Image Below To Answer The Identify The Following. Please Show Ur Work Bc I Have Too !!One Pair

Answers

Answer 1

∠1 and ∠4 are vertical angles

∠1 and ∠3 are adjacent angles

∠1 and ∠2 are supplementary angles

Vertical Angles:

Vertically opposing angles are established when two lines intersect at a single place and the angles opposite to each other are created using the two intersecting lines. There is never a difference between these angles.

Adjacent Angles:

Those angles are referred to as vertical angles if they share an arm and a vertex. Additionally, the nearby angles are those that are produced side by side. The primary characteristic of these angles is that they never cross over.

Supplementary Angles:

When two angles add up to 180 degrees, they are said to be supplementary. Angles are referred to be supplementary angles when two of them are next to one another.

To know more about Angles:

https://brainly.com/question/28451077

#SPJ1


Related Questions

What is the gradient of the
line that passes through the
points (0,0) and (3, 12)?
m=
Submit Answer

Answers

Answer:

4

Step-by-step explanation:

gradient = slope = m = rise / run

  from the origin (0,0) this line rises '12' and has a run of '3'

             12/3 = 4

suppose a 90% confidence interval for the population mean resulted in an upper limit of $658 and a lower limit of $273. what is the correct interpretation of this confidence interval?

Answers

This confidence interval suggests that we are 90% confident that the population mean lies between $273 and $658.

This confidence interval provides us with an estimate of the population mean with 90% confidence. This confidence interval suggests that we are 90% confident that the population mean lies between $273 and In other words, we can be 90% sure that the population mean falls within this range. This confidence interval does not tell us the actual population mean, but it provides us with an estimate of where the population mean is likely to lie. Furthermore, it also provides us with an indication of the degree of uncertainty associated with our estimate.

Learn more about population mean here

https://brainly.com/question/19538277

#SPJ4

how would you change the test procedure of part (b) to obtain a test with significance level .05? what impact would this change have on the error probability of part (c)?

Answers

the impact on the error probability of part is on  Decreasing trends of the test procedure

A Type II error is when we fail to reject a false null hypothesis. Higher values of α make it easier to reject the null hypothesis, so choosing higher values for α can reduce the probability of a Type II error.

The consequence here is that if the null hypothesis is true, increasing α makes it more likely that we commit a Type I error (rejecting a true null hypothesis).

So using lower values of α can increase the probability of a Type II error.

learn more about of error here

https://brainly.com/question/15582747

#SPJ4

If the simple interest on 4000$ for 6 years is 1440$ then, what is the interest rate?

Answers

The rate of interest  for the given data can be found as 6%.

What is simple interest?

Simple interest can be defined as a form of interest in which the percent rate is applied on the same principal for a given period of time. The amount can be calculated by adding the interest to the principal.

The given data as per the question is as below,

P = $4000, t = 6 years and Interest = $1440.

Suppose the rate of interest be r% per annum.

Substitute all the values in the expression for simple interest (P × r × t)/100 as follows,

1440 = (4000 × 6 × r)/100

⇒ r = 6%

Hence, the rate of interest is obtained as 6%.

To know more about Simple interest click on,

https://brainly.com/question/25845758

#SPJ1

4. Shoan takes 5 hours to complete 4 assignments. Find the
time taken to complete 3 assignments.

Answers

Answer:

3.75 Hours or 225 Minutes

Step-by-step explanation:

5/4 = 1.25 So it takes 1.25 hours to do an assignment

1.25 x 3 = 3.75

Which equals 225 Minutes

Answer: 3.75 or 3 3/4

Step-by-step explanation:

This is a unit rate problem

A unit rate is a rate for one of something. (i.e, 1 pencil for 4 notebooks)

So first, to find the time needed for one assignment, we divided 5 hours by 4 hours.

5/4 = 1.25 (1 and 1/4)

Now we multiply the unit rate by 3.

1.25 times 3 = 3.75, or 3 3/4.

The figure shows two vectors T⃗ and U⃗ separated by an angle θTU.You are given that T⃗ =(3,1,0), U⃗ =(2,4,0), and T⃗ ×U⃗ =V⃗ .1) Express V⃗ as an ordered triplet of values, separated by commas.2) Find the magnitude of V3) Find the sine of the angle between T⃗ and U⃗ .

Answers

The [tex]\vec{v}[/tex] as an ordered triplet of values, separated by commas is ( 0 , 0 , 10 )

The magnitude of V is 10 units.

The sine of the angle between  [tex]\vec{T}[/tex] and [tex]\vec{U}[/tex] is [tex]45^{\circ}[/tex].

As per the given data the value of [tex]\vec{T}[/tex] is ( 3, 1, 0 ) and [tex]\vec{U}[/tex] is ( 2, 4, 0 )

Firstly we have to determine the value of [tex]\vec{v}[/tex].

The value of [tex]\vec{v}[/tex] is given by [tex]\vec T \times \vec U[/tex]

[tex]& v=\left|\begin{array}{lll}\hat{i} & j & \hat{k} \\3 & 1 & 0\\2 & 4 & 0\end{array}\right|[/tex]

[tex]=\hat{k}(12-2)[/tex]

[tex]=10 \hat{k}[/tex]

Therefore [tex]\vec{v}[/tex] = ( 0 , 0 , 10 )

Now we have to determine the value of magnitude of V.

Magnitude of V is represented by [tex]$|\vec{v}|[/tex]

[tex]$|\vec{v}|=\sqrt{100}[/tex]

[tex]$|\vec{v}|[/tex] = 10 units

Therefore the magnitude of V is 10 units.

Now we have to determine the value of sine of the angle between  [tex]\vec{T}[/tex] and [tex]\vec{U}[/tex].

[tex]|\vec{v}|=|\tilde{T}||\tilde{U}| \sin \theta_{T U}[/tex]

[tex]\Rightarrow \theta_{T U}=\sin ^{-1}\left(\frac{|\vec{v}|}{|\vec{T}| |\vec{U}|}\right)[/tex]

[tex]=\sin ^{-1}\left(\frac{10}{\sqrt{200}}\right)[/tex]

[tex]=45^{\circ}[/tex]

Therefore value of sine of the angle between  [tex]\vec{T}[/tex] and [tex]\vec{U}[/tex] is [tex]45^{\circ}[/tex]

For more questions on vectors

https://brainly.com/question/25526667

#SPJ4

calculate the double integral. 3x sin(x + y) da, r = 0, π 6 × 0, π 3 r

Answers

The value of the double integral ∫∫3x sin(x + y) dA, R = [0, π/6] × [0, π/3] is  0.677

In this question we need to calculate the double integral. 3x sin(x + y) dA, R = [0, π/6] × [0, π/3]

i.e., to find ∫∫3x sin(x + y) dA, R = [0, π/6] × [0, π/3]

First we integrate the function 3x sin(x+y) as a function of y, treating the variable x as a constant.

G(x) = ∫_[0, π/6] (3x sin(x+y)) dy

G(x) = 0.402 xcos(x) + (3x sin(x) / 2)

Now we calculate the integral of the previous result as a function of x.

i.e., ∫_[0, π/3] G(x) dx

= ∫_[0, π/3] [0.402 xcos(x) + (3x sin(x) / 2)] dx

= 0.677

Therefore, the solution to the double integral. 3x sin(x + y) da, r = [0, π/6] × [0, π/3] is  0.677

Learn more about the double integral here:

https://brainly.com/question/15072988

#SPJ4

Find the gradient vector field of f and sketch it:
f(x,y)= √(x2+y2)

Answers

The gradient vector field of f is ∇(f) = (xi+yj)/√(x^2+y^2) ans sketch is given below.

In the given question, we have to find the gradient vector field of f and sketch it.

f(x,y)= √(x^2+y^2)

To find the gradient, it's just the partial derivatives for x and y

∇(f) = ∂f/∂x i + ∂f/∂y j

In this case, we have

∂f/∂x = x/√(x^2+y^2) and ∂f/∂y = y/√(x^2+y^2). So

∇(f) = x/√(x^2+y^2) i + x/√(x^2+y^2) j

We can write it as

∇(f) = (xi+yj)/√(x^2+y^2).

This is a unitary vector field everywhere, with the exception of the origin, where it is undefined, where we can see it as the vector xi+yj divided by the magnitude √(x^2+y^2). From the starting point, the direction is outward.

The sketch is given below:

To learn more about gradient vector field link is here

brainly.com/question/29815461

#SPJ4

The system of equations is graphed on the
coordinate plane.
y=-1/2x-1
y=1/4x-4
Enter the coordinates of the solution to the system of
equations in the boxes.

Answers

Answer:

[tex](4,-3)[/tex]

Step-by-step explanation:

The solution is where the graphs intersect.


Which function models the data in the table below?



y = -1/2x + 5

y = -2x + 5

y = -1/2x + 10

y = -2x + 10

Answers

The function that models the data in the given table is: D. y = -2x + 10.

How to Write the Function that Models the Data in a Table?

The function that models the data of a table can be expressed in the slope-intercept form as y = mx + b, where:

the slope = m = change in y / change in x.y-intercept = b.

Find the slope of the function using two pairs of values from the table, (6, -2) and (10, -10):

Slope (m) = change in y / change in x = (-10 - (-2)) / (10 - 6)

Slope (m) = -8 / 4

Slope (m) = -2

Find the y-intercept (b) by substituting m = -2 and (x, y) = (6, -2) into y = mx + b:

-2 = -2(6) + b

-2 = -12 + b

-2 + 12 = b

10 = b

b = 10

To write the function, substitute m = -2 and b = 10 into y = mx + b:

y = -2x + 10

Learn more about the function of a table on:

https://brainly.com/question/26507192

#SPJ1

where s is weekly sales in dollars and t is the number of weeks since the end of the campaign. (a) find the rate of change of s (that is, the rate of sales decay).

Answers

The rate of change of S for the weekly sales is :

-250 000e⁻°·⁵t

Given:

the sales of the product is :

S = 500,000e^-0.5t

where S is the weekly sales in the dollars and t is the number of weeks since end of the campaign.

we are asked to find the rate of sales dS/dt = ?

that is the rate of sales of delay.

Given:

S = 500,000e^-0.5t

dS/dt =(500,000)e^-0.5t + 500,00 (e^-0.5t)

= 0 +  500,00 (e^-0.5t)

= - 250 000 e^0.5t

therefore, dS/dt = - 250 000 e^0.5t

Hence we get the required answer.

Learn more about Derivatives here:

brainly.com/question/12047216

#SPJ4

Find the equation of the straight line that has a slope of 4 and passes through the point (-1, -6)

Answers

Answer:

y = 4x - 2

Step-by-step explanation:

y = 4x + b

-6 = 4(-1) + b

-6 = -4 + b

b = -2

y = 4x - 2

Answer:

[tex]y - (-6) = 4(x - (-1))[/tex] or [tex]y = 4x - 2[/tex]

Step-by-step explanation:

We can use point slope form: [tex]y - y_{1} = m(x - x_{1})[/tex], where [tex](x_1, y_1)[/tex] is a point on the line and [tex]m[/tex] is the slope.

We plug in (-1, -6) and 4 to get:  which simplifies to [tex]y + 6 = 4(x + 1)[/tex]

If we want to express this in the form y = mx + b, we have to do a little rearranging. [tex]y + 6 = 4(x + 1)[/tex] becomes [tex]y + 6 = 4x + 4[/tex] and then we subtract 6 from both sides to get: [tex]y = 4x - 2[/tex]

Fred works for $11.25 per hour. His regular hours are 40 hours per week and he gets time and a half for overtime. One week he works 48.5 hours. He wants to calculate his pay. Complete the table.


rate: $11.25, regular hours:--------------, overtime: N/A, total: $----------

rate: $--------, regular hours: N/A, overtime: ---------, total: $----------

gross: $----------

Answers

Fred's pay for the specified week, with a rate of $11.25 for regular hours and time and a half for overtime, and number of hours worked of 48.5 hours is presented by completing the attached table as follows;

Rate: $11.25, regular hours: 40, overtime: N/A, total: $450

Rate: $16.875, regular hours: N/A, overtime: 8.5, total: $143.4375

Gross: $593.4375

What is a payment rate?

A payment rate is the amount of payment or money that is received in a unit of time.

The amount Fred earns per hour = $11.25

The number of regular hours Fred works per week = 40

The rate at which he is paid for overtime = Time and a half

The number of hours Fred works in the specified week = 48.5 hours

Therefore, the amount Fred earns for working overtime = 1.5 × $11.25 = $16.875

The amount Fred earns for regular hours during the specified week = 40 × $11.25 = $450

The number of overtime hours Fred works for during the week = 48.5 - 40 = 8.5

The amount Fred earns for the overtime work = 8.5 × $16.875 = $143.4375

Fred's gross payment for the week is therefore; $450 + $143.4375 = $593.4375

The table can therefore be completed by plugging in the above values as follows;

Rate: $11.25, regular hours: 40, overtime: N/A, total: $450

Rate: $16.875, regular hours: N/A, overtime: 8.5, total: $143.4375

Gross: $593.4375

Learn more more about gross pay calculations here:

https://brainly.com/question/11256904

#SPJ1

ZA and ZB are vertical angles. If mA = (5x + 5)° and mZB = (6x − 8),
then find the value of x.

Answers

Answer:

x = 13

Step-by-step explanation:

vertical angles are congruent , then

∠ B = ∠ A , that is

6x - 8 = 5x + 5 ( subtract 5x from both sides )

x - 8 = 5 ( add 8 to both sides )

x = 13

Asher is 12 years old and this season. They joined a traveling baseball team for players 17 years old and under. Asher hit 13 doubles this year.

If Asher plays each of the remaining seasons before being too old, what is the average number of doubles Asher needs to hit in order to match the league record of 73 career doubles?

___ doubles (per season)

Answers

Answer:

3

Step-by-step explanation:

His total doubles 73 and he already has 13 doubles. So, 73-13 = 60.He needs 60 doubles. His current age is 12. So, 17-12 = 5 yrs. 1 yr has 4 season. For 5 yrs 20 season. 60/20 = 3. Hence 3 doubles/season

by sethus1981.

Add 8x-9y+2z , -6x +7y-5z

Answers

Answer:

2x-2y-3z

Step-by-step explanation:

8x+-6x

=8x-6x

=2x

-9y+7y

=-2y

2z+-5z

= 2z-5z

=-3z

2x-2y-3z

8x - 9y + 2z

-6x + 7y - 5z

On adding these two equation :

8x + ( -6x) = 2x

-9y + 7y = -2y

2z + (-5z) = -3z

we will get :

2x - 2y -3z

.

In the addition/subtraction method, the two equations in the system are added or subtracted to create a new equation with only one variable. In order for the new equation to have only one variable, the other variable must cancel out. In other words, we must first perform operations on each equation until one term has an equal and opposite coefficient as the corresponding term in the other equation.

.

To learn more about addition/subtraction method of equations

Here :

https://brainly.com/question/11958350

A, B & C form the vertices of a triangle, where
∠CAB = 90°.
AB = 10.2 m and AC = 4.9m. Evaluate
∠ ACB, giving your answer rounded to 3 SF.

Answers

Answer:

Step-by-step explanation: To find the measure of angle ACB, we can use the Pythagorean Theorem: AB^2 + AC^2 = BC^2.

Substituting in the given values for AB and AC, we get: 10.2^2 + 4.9^2 = BC^2

Solving for BC, we get: BC = √(10.2^2 + 4.9^2)

Substituting this value back into the equation for the Pythagorean Theorem, we get:

AB^2 + AC^2 = (√(10.2^2 + 4.9^2))^2

Simplifying this equation, we get:

AB^2 + AC^2 = 10.2^2 + 4.9^2

This equation is already in the correct form, so we can solve for ∠ACB directly:

∠ACB = tan^-1(AC/AB)

Substituting in the given values for AC and AB, we get:

∠ACB = tan^-1(4.9/10.2)

Using a calculator, we find that tan^-1(4.9/10.2) is approximately equal to 26.7°.

Rounded to 3 significant figures, this value is 26.7°.

The question is below, please answer. This is in a hurry!

Answers

Answer:

A)

Step-by-step explanation :

This question isn't too hard, but you would have to use similar triangles.

Michael’s class grew plants as part of the experiment. Michael’s plant grew 1/12 inch in the first week, 3/12 of an inch in the second week, and 5/12 of an inch in the third week. How much did Michael’s plant grow in those three weeks?

Answers

Answer:

{ No SIMPLIFIED: [tex]\frac{9}{12} inches[/tex] }
{ SIMPLIFIED: [tex]\frac{3}{4} inches[/tex] }

Step-by-step explanation:

If we want to know about the plant grows, we need to add the fractions in 3 weeks with this solution:
[ The inch of the plant 1st week ] + [ The inch of the plant 2nd week ] + [ The inch of the plant 3rd week ] = [ The inch of the plant in 3 weeks ]

Look at the denominator of 3 weeks, it is 12 and the numerators are 1,3,5
To plus the fraction we remain the denominator and plus the numerators together which we can result in this equation:
[tex]\frac{1}{12}[/tex] + [tex]\frac{3}{12}[/tex] + [tex]\frac{5}{12}[/tex] = [tex]\frac{1 + 3 + 5}{12}[/tex] = [tex]\frac{9}{12}[/tex] { SIMPLIFIED: [tex]\frac{9 : 3 }{12 : 3} = \frac{3}{4}[/tex] }
So then we see that if you choose no simplified then: Michael's plant grow  [tex]\frac{9}{12}[/tex] inches in 3 weeks or you could choose the simpliflied one then: Michael's plant grow [tex]\frac{3}{4}[/tex] inches in 3 weeks.

For more answers, Contact:
https://brainly.com/app/profile/63103549/answers

11. Simplify the expression.
2x - 5(3x-4)

Answers

Answer:

Your answer is -13x + 20

Step-by-step explanation:

2x - 5*(3x - 4)

Discribute the -5.

2x + [ (-5)(3x) + (-5)(-4) ]

2x + [ (-15x +20) ]

2x - 15x + 20

- 13x + 20

m and p are in direct proportion.
The equation of proportionality is m = 8p.
If p increases from 3 to 7, how much will m increase by?
If your answer is a decimal, give it to 1 d.p.

Answers

M will increase from 24 to 56

What is meant by direct proportion ?

The relationship between two values where their ratio equals a constant number is known as a direct proportion or direct variation. It is symbolised by the proportional sign ().

According to the direct proportion formula, if y is directly proportional to x, we can state that y = kx, where k is a constant. Y = kx also serves as the direct proportion equation's general form.

When one quantity increases or declines, the other quantity also rises or falls in direct proportion. On the other hand, in indirect or inverse proportion, if one quantity rises, the other one falls, and vice versa.

m= 8p

When p = 3        

 m =3 × 8 =24

When p = 47      

  m=8× 7 = 56

So M will increase from 24 to 56

To learn more about direct proportion refer to :

https://brainly.com/question/1266676

#SPJ1

in the scientific method, which of the following is true about a hypothesis? group of answer choices the same hypothesis may not be tested more than once. research studies are designed to prove a hypothesis. a specific hypothesis is generated based on an established theory. a hypothesis both explains and predicts a phenomenon.

Answers

For the scientific method , the true statement about the hypothesis is  (b) research studies are designed to prove a hypothesis.

The term Hypothesis is defined as the observation which is proposed as the possible outcome or results of an experiment .

we know that ; in order to prove a hypothesis, the experiments are designed and after that they are performed based on given hypothesis.

That is how the experiments are designed based on a previous idea which may result in a particular outcome.

So , any hypothesis is dependent on a specific experiment( research studies) .

Therefore , the correct option is (b) .

The given question is incomplete , the complete question is

In the scientific method, which of the following is true about a hypothesis?

(a) the same hypothesis may not be tested more than once.

(b) research studies are designed to prove a hypothesis.

(c) a specific hypothesis is generated based on an established theory.

(d) a hypothesis both explains and predicts a phenomenon.

Learn more about Hypothesis here

https://brainly.com/question/27982373

#SPJ4

determine if the line integral of the vector field sketched in the figure along the curve c is positive, negative, or zero.

Answers

The line integral of the vector field in the figure is zero. This is due to the fact that the vector field is perpendicular to the curve c, and therefore, the integral of the dot product of the vector field and the tangent vector along the curve c is zero.

1. The line integral of a vector field is the integral of the dot product of the vector field and the tangent vector of the curve along the curve.

2. The vector field in the figure is perpendicular to the curve c.

3. The dot product of two perpendicular vectors is zero.

4. Therefore, the integral of the dot product of the vector field and the tangent vector along the curve c is zero.

5. Hence, the line integral of the vector field in the figure is zero.

Learn more about integral here

https://brainly.com/question/18125359

#SPJ4

Help and explain to me how pls and thank you

Answers

The two sequence A and sequence B both are correct.

What is Graph?

Graph is a mathematical representation of a network and it describes the relationship between lines and points.

Given that triangle UVW and Triangle ABC are congruent.

A transformation is a way of changing the size or position of a shape.

Every point in the image is the same distance from the mirror line as the original shape.

Triangle UVW made a 180 degrees of rotation about point V and then a triangle along the directed line segment VB.

This is same as translation along the directed line segment UA then about 180 degrees rotation about point A.

Hence, the two sequence A and sequence B both are correct.

To learn more on Graph click:

https://brainly.com/question/17267403

#SPJ1

Kenji and Ramon are running cro country. Kenji run at a rate of 150 meter per minute

Answers

Kenji and Ramon are at a distance of 750 m after a time interval of 1 hour from each other.

Kenji runs for a total time of 60 minutes + 15 minutes = 75 minutes.

Ramon runs for a total of 60 minutes.

Now, we find the total distance covered by Kenji and Ramon in the time intervals of 75 minutes and 60 minutes respectively.

Kenji runs a total distance of 150 meters/minute * 75 minutes = 11250 meters.

Ramon runs a total distance of 200 meters/minute * 60 minutes = 12000 meters.

Now, we find the difference in their distances.

The distance between Kenji and Ramon is 12000 meters - 11250 meters = 750 meters.

Learn more about Distance at:

brainly.com/question/26046491

#SPJ4

Complete Question:

Kenji and Ramon are running cross country. Kenji run at a rate of 150 meter per minute and Ramon runs at a rate of 200 meters per minute. If Kenji starts 15 minutes before Ramon, how far apart are they after one hour?

solve by graphing. please show work

Answers

The graph is a parabola with its touching the x-axis at (3,0)

What is a parabola?

A parabola is a curve where any point is at an equal distance from:

a fixed point (the focus ), and

a fixed straight line (the directrix )

its general equation is y = a(x – h)² + k or x = a(y – k)² +h. Here (h, k) denotes the vertex. y = a(x – h)2 + k is the regular form. x = a(y – k)2 +h is the sidewise form.

Given here (x-3)² + 4 = -4/9(x(x-6))

simplifying this we get

9(x²-6x+13) = -4 (x(x-6))

13x²-78x+117 = 0

(x-3)² = 0

thus it has a root at x=3

The graph is attached showing the parabola  touching the x-axis at (3,0)

Learn more about parabola here:

https://brainly.com/question/21685473

#SPJ1

Which of the following statements is true about similar triangles? *
A. The measurements of corresponding angles differ between the two triangles.
B. Their corresponding angles are congruent, and the corresponding sides are proportional.
C. If two triangles are similar, both their corresponding sides and angles are always congruent.

Answers

Two triangles are similar when their corresponding angles are congruent, and the corresponding sides are proportional. Hence, option C s the most appropriate answer to the given question.

When we are given triangles, we can prove that two triangles are similar to the given conditions by applying various tests of similarity.

Similar triangles are triangles that have the same shape, but they may have different sizes.

Hence, to prove them to be similar, we have to prove the corresponding angles are congruent and the corresponding sides are proportional. Hence, option C is the only right choice for the given question.

Read more about Similar triangles:

brainly.com/question/11920446

#SPJ4

Factor X^3-2x^2-8x???

Answers

Answer:

x( x-4)(x+2)

Step-by-step explanation:

x^3-2x^2-8x

First factor out the greatest common factor x

x( x^2 -2x -8)

What 2 numbers multiply to -8 and add to -2

-4*2 = -8

-4+2 = -2

x( x-4)(x+2)

-1/2p - 4< 3

Please try to graph on number line

Answers

We can begin by rewriting the problem:

3/4 - 1/2p > 5/4

Let’s solve accordingly through steps:

1. Subtract 3/4

-1/2p > 2/4

2. Divide both sides by the coefficient

-1/2 / -1/2 = p

2/4 / -1/2 = -1

3. Simplify & Switch Signs

p < -1

The answer to Part A is p < -1.

Part B: On a number line, this answer would be graphed by plotting a point at -1 and drawing an arrow that goes to the left direction to show that p is less than -1.

Плиз сделайте даю 20 баллов

Answers

Answer:

sorry I don't know now how to read

Other Questions
James Fennimore Cooper authored "The last of the Mohicans" ina. 1824b. 1912c. 1826d. 1927 which activity would a general registered nurse be expected to perform as part of genomic care? The table of values represents a proportional relationship.What is the constant of proportionality in the relationship, written as an improper fraction (fraction greater than one)?X Y2 36 9 the primary question to answer as part of the economic feasibility assessment is: group of answer choices should we build it? when and where will it be used? if we build it will they use it? can we build it? How did this amendment signal a shift away from Presidential Reconstruction?A. The law authorized federal troops to enforce their plan for Reconstruction in the South.B. The law extended the protections of the Civil Rights Act of 1866, which Johnson had vetoed.C. The law ensured that black men would have the right to vote and influence Reconstruction policies.D. The law overturned Black Codes in the South, which were part of Johnsons Reconstruction plan.Use the Fourteenth Amendment to answer the question.Section 1.All persons born or naturalized in the United States, and subject to the jurisdiction thereof, are citizens of the United States and of the State wherein they reside. No State shall make or enforce any law which shall abridge the privileges or immunities of citizens of the United States; nor shall any State deprive any person of life, liberty, or property, without due process of law; nor deny to any person within its jurisdiction the equal protection of the laws.Section 2.Representatives shall be apportioned among the several States according to their respective numbers, counting the whole number of persons in each State, excluding Indians not taxed. But when the right to vote at any election for the choice of electors for President and Vice-President of the United States, Representatives in Congress, the Executive and Judicial officers of a State, or the members of the Legislature thereof, is denied to any of the male inhabitants of such State, being twenty-one years of age, and citizens of the United States, or in any way abridged, except for participation in rebellion, or other crime, the basis of representation therein shall be reduced in the proportion which the number of such male citizens shall bear to the whole number of male citizens twenty-one years of age in such State.Section 3.No person shall be a Senator or Representative in Congress, or elector of President and Vice-President, or hold any office, civil or military, under the United States, or under any State, who, having previously taken an oath, as a member of Congress, or as an officer of the United States, or as a member of any State legislature, or as an executive or judicial officer of any State, to support the Constitution of the United States, shall have engaged in insurrection or rebellion against the same, or given aid or comfort to the enemies thereof. But Congress may by a vote of two-thirds of each House, remove such disability.Section 4.The validity of the public debt of the United States, authorized by law, including debts incurred for payment of pensions and bounties for services in suppressing insurrection or rebellion, shall not be questioned. But neither the United States nor any State shall assume or pay any debt or obligation incurred in aid of insurrection or rebellion against the United States, or any claim for the loss or emancipation of any slave; but all such debts, obligations and claims shall be held illegal and void.Section 5.The Congress shall have the power to enforce, by appropriate legislation, the provisions of this article. FILL IN THE BLANK. The period of a sine or cosine function is given by ______. phase shift. For the function given by y = sin(bx-c), c/b represents the ______ ______ of the ... mary is a split-brain patient, which means that she . a. has an underdeveloped cerebellum b. has motor impairments c. has damage to the prefrontal cortex d. had her corpus callosum severed Often, weather forecasters refer to high- or low-pressure areas movingtoward particular regions in order to describe predicted conditions forthose areas. As air rises in low-pressure systems, it cools and oftencondenses into clouds and precipitation, resulting in storms. Inhigh-pressure systems, the air sinks toward the Earth and warmsupward, leading to dry and fair weather.HOW DOES THIS RELATE TO MY RESEARCH QUESTION WHICH IS WHAT EFFECT DOES THE AMOUNT OF AIR PRESSURE HAVE ON HOW FAR A SOCCER BALL TRAVELS WHEN KICKED? find the nth maclaurin polynomial for the function f (x )space equals space sin space x comma space space n space equals space 5. the coefficient of x to the power of 5 is For this philosopher, the existence of human beings is not reduced to the realization of an essence thought by God; man exists to the extent that he is carried out, it is the set of his acts and nothing else. There is a trend in many industries to hire more _____ than _____.Full-time employees; freelancersvolunteers; experts freelancers; full-time employees experts; volunteers nrg inc. currently has $331 million of debt outstanding. it's cost of debt is 5.5% and its marginal corporate tax rate is 25%. estimate the tax shield for nrg at the end of the year (don't calculate the present value). express your answer in $-millions and round to two decimals (do not include the $-sign in your answer). Find an equation of the tangent to the curve at the given pointx=cos(t)+cos(2t)y=sin(t)+sin(2t)(-1,1)y=? here is a small part of the order book for mesquite foods: bidask pricesizepricesize $ 103100$ 103.50200 $ 102.50200$ 103.80200 $ 101400$ 104300 $ 99.80300$ 104.50400 georgina sloberg submits a market order to sell 100 shares. what price will she receive? norman pilbarra submits a market order to buy 400 shares. what is the maximum price that he will pay? note: round your answer to 2 decimal places. a connective statement in the middle of a speech that explains what you are about to discuss is called an internal what is one characteristic of functional programming languages that makes their ssemeantics ismpler than that of imperative languages? This theory gives greater recognition to the importance of our interpersonal interactions in shaping the aging experience.Select one:a. social constructionismb. disengagement theoryc. social phenomenologyd. political economy theory Overtime Hours Worked A random sample of 15 registered nurses in a large hospital showed that they worked on average 44.6hours per week. The standard deviation of the sample was 2.3. Estimate the mean of the population with 99% confidence.Assume the variable is normally distributed. Round intermediate answers to at least three decimal places. Round your finalanswers to one decimal place.06 Please please tell me what is the main topic of this passage and how do you know this Historia help again :/:/:/:/:/:/